0% found this document useful (0 votes)
45 views32 pages

Worksheet 2 Answer

The passage discusses how the development of photography changed people's views of art and nature. Photography led people to realize that merely depicting existing works was not art, and influenced how people saw and portrayed nature in paintings. It also describes how photographs presented an unfiltered perspective of the world that affected perception.

Uploaded by

Himadri Badoni
Copyright
© © All Rights Reserved
We take content rights seriously. If you suspect this is your content, claim it here.
Available Formats
Download as PDF, TXT or read online on Scribd
0% found this document useful (0 votes)
45 views32 pages

Worksheet 2 Answer

The passage discusses how the development of photography changed people's views of art and nature. Photography led people to realize that merely depicting existing works was not art, and influenced how people saw and portrayed nature in paintings. It also describes how photographs presented an unfiltered perspective of the world that affected perception.

Uploaded by

Himadri Badoni
Copyright
© © All Rights Reserved
We take content rights seriously. If you suspect this is your content, claim it here.
Available Formats
Download as PDF, TXT or read online on Scribd
You are on page 1/ 32

© Copyright 2020 @LEGALIGHT Education

LOGICAL REASONING (Sectional Test 4) PAGE 1


© Copyright 2020 @LEGALIGHT Education

Duration: ……. Mins DATE………………………………………


No. of Questions: 142 YOUR TIME………………………………

Passage 1
1. D
A: While the author does state that photography was mocked at first (paragraph 1), the author never suggests that
photography was eventually accepted by the public as an art form. Rather, the passage discusses how photography
led people to realize that some works they had previ- ously considered to be art were really just pictures of works of
art, not artistic creations in their own right (see paragraph 2).
B: This choice, while supported by the passage (paragraph 2), is too narrow to be the primary purpose of the passage.
Most importantly, it leaves out the author’s discussion of how photog- raphy affected how people viewed and
perceived nature (paragraph 3).
C: The second part of this choice is not supported by the passage. While the etcher Jacquemart’s work was devalued
(“the ground was cut from under [his] feet” [paragraph 2]) due to revela- tions brought about by photography, the
author does not indicate that this was unfair or “unfortunate.” In fact, the author suggests that he agrees with those
who “realized” that Jacquemart was “merely a reporter of works of art and not a maker of them” (paragraph 2). The
passage also does not suggest that reevaluation and devaluation of any other artists was unjust, nor does the author
use a negative tone in the second paragraph. Finally, this choice does not include the main idea of the third paragraph,
which is an important theme in the passage.
D: Yes. The first and second paragraphs discuss how photography changed how the public views and evaluates both
“pictorial expression” and “pictorial communication of statements of fact.” The third paragraph explains how
photography “conditioned” people to “think photographically, and thus to see for themselves things that previously it
had taken the photograph to reveal”. That is, photography changed how people saw nature, or reality, it- self. In this
paragraph the author also states that “Just as nature had once imitated art, now it began to imitate the picture made
by the camera”. That is, our perception of nature was affected by the depiction of nature, or reality, in photographs.
(Note that the author uses “nature” in a general way, to refer to “the appearance of everything”.) Finally, the author
sees the effects of photography as “a permanent gain”, which supports the positive tone of this answer choice.

2. A
A: Yes. A main theme of the passage is that once it was realized that a photograph could better depict “the quality and
character of something” (paragraph 2), people came to distinguish between skilled representations of objects on one
hand, and works of art on the other.
Therefore, we can infer that the author of the passage might agree with a statement that discusses the necessary
conditions for creating art, in this case artistic photographs, and so suggests that not all photographs qualify as art.
Notice the moderate wording of this answer choice; we can infer that the author would agree that the statement in
the ques- tion stem “may be true,” even if we don’t know for sure that the author would entirely or definitively agree.
Finally, there is no other answer choice that is better supported by the passage.
B: While the passage does say that eventually photography was common, it also states that this happened slowly
(paragraph 1). Therefore, the passage does not suggest that the technical as- pects can be quickly mastered.
Furthermore, Stieglitz’s statement is about taking photographs that qualify as art, not simply the capacity to take a
photograph (i.e., to master the technical aspects). Thus there is no reason, based on the passage, to believe that the
author would be skeptical on this basis.
C: The author never discusses photography as an art form, nor does he describe any photographs as “artistic” (while
he also does not rule out the possibility that they could be art). The pas- sage in large part describes how the fact that
photographs could so accurately depict works of art made the public realize that depictions of works of art (including
those made by photog- raphers) do not qualify as art. Therefore, we cannot conclude that Stieglitz’s statement is, or
would be accepted by the author as, a natural continuation of the author’s own analysis.
D: The author does in fact discuss “qualities of artistry” to the extent that some things do not qualify as art (second
and third paragraphs).

3. B
A: The author never discusses art’s fundamental goal, only that certain pictorial representations do not qualify as art.

LOGICAL REASONING (Sectional Test 4) PAGE 2


© Copyright 2020 @LEGALIGHT Education
B: Yes. See paragraph 3. The author states that as people became accustomed to the view of nature shown by
photographs, they “began to think photographically, and thus to see for themselves things that it had previously taken
the photograph to reveal to their astonished and protesting eyes.” The author also states, “Just as nature had once
imitated art, now
it [nature, meaning our perception of nature] began to imitate the picture made by the camera”. These lines indicate
that viewing photographs changed how people viewed the world around them. Finally, the author then states that
“willy nilly many of the painters began to follow suit,” meaning that this new view of the world then affected how
painters depicted or represented nature. Note that the author does not use “nature” in a way that is limited to certain
parts of the world (trees, wild animals, etc.). Rather, he uses it as a syn- onym for the world around us, or “the
appearance of everything”.
C: It was photographs, not theoretical discussions, which changed how we perceive and repre- sent nature (see the
explanation for choice B). This choice is “partially correct”; everything
but “theoretical discussions” is fine, but those words are enough to make the choice as a whole incorrect.
D: This choice is too extreme. While the author does indicate that the work of some (e.g., Jacquemart) who
represented works of art (that is, represented objects that represent nature) came to be less valued than it was
previously (paragraph 2), this is not enough to show that artistic creation is always more highly valued. Furthermore,
this choice does not deal with the issue of the question, which is the relationship between the depiction of nature and
the perception of nature itself.

4. C
A: Be careful not to use outside knowledge or to interpret the cited words out of the context of the passage. The
passage never discusses how photographers might purposefully or consciously manipulate images.
B: This choice is too extreme. The author states that “many of the painters began to follow suit” (paragraph 3); that is,
painters began to depict reality in a way that imitated photographs. This suggests that some paintings do distort the
picture in the same way as a photograph would.
C: Yes. The author states that photographic distortion “only meant that the camera had not been taught, as human
beings had been, to disregard perspective in most of its seeing” (paragraph 3). This indicates than when we see things
directly, without the mediation of a photographic image, we interpret the relative physical relationship between
objects (per- spective) in a way that a photograph does not.
D: Be careful not to use outside knowledge or to take the words of the passage out of context.
The passage describes “distortion” as something inherent to the nature of a photograph (para- graph 3), not as a side
effect of primitive or inadequate technology.

5. A
I: True. See paragraph 2. The author suggests that a new distinction came to be drawn between artists and non-artists.
Jacquemart was now shown to be “merely a reporter of works of art” and was no longer seen as an artist. By using
changing public opinion about Jacquemart to illustrate how a new distinction came to be drawn between “pictorial
expres- sion [art] and pictorial communication of statements of fact [not art],” the author indicates that Jacquemart
was once seen as a good artist. Note that the distinction between good and poor artists was, before photography,
drawn on the basis of skill, and that Jacquemart is described as highly skilled.
II: False. See paragraph 2. The word “only” makes this choice incorrect. While it is true that pho- tographs were more
accurate, this is not the only reason his work was devalued. Rather, the fact that photographs were so accurate led to
a new distinction being drawn between art and representations of art. It was this new distinction or perception that
was the main cause of the change in perception of Jacquemart’s work.
III: False. See paragraph 2. Note the words “not only and “but also”. This transition tells us that Jacquemart was not a
“humble workaday factual illustrator.” While it is possible, only given that wording, that he was some other kind of
illustrator than a “humble workaday” one, the description given of his work gives no indication that he made
illustrations, or that his reputa- tion was one of an illustrator. Instead, the passage suggests that he was once seen as
an artist; artists and illustrators are not equated in the passage.

Passage 2
6. C

LOGICAL REASONING (Sectional Test 4) PAGE 3


© Copyright 2020 @LEGALIGHT Education
A: This would be a too literal reading of that phrase. The passage does not indicate that Napoleon actually took the
throne, only that he “behaved” like a king while ruling Portugal. Further- more, there is no indication that Napoleon,
while behaving like a king, claimed that his rule was sanctioned by God or the Church (“divine rights of a monarch”).
B: While the passage indicates earlier (paragraph 3) that the Portuguese monarchy restricted these rights, the author
never argues that all or most traditional orders do so. Also, the issue in this part of the passage is the specific behavior
of Napoleon. The passage does not suggest that Napoleon is a traditional ruler, nor does it generalize from his behavior
to that of other rulers.
C: Yes. This part of the passage contrasts the hopes of the Portuguese liberals with the reality of Napoleonic rule. By
saying that the liberals flocked to Napoleon with the hope of repre- sentative government, and yet that Napoleon
“humored” them, “secretly laughed at them” and then behaved like a king (and monarchies are not representative
regimes, as we know from paragraph 3), it suggests that the liberals’ hopes were dashed.
D: There is no indication that Junot intended to institute a representative government, or that Napoleon’s and Junot’s
goals differed. There is also no indication that Napoleon at any point acted against Junot’s wishes, or vice versa. The
passage describes them both as laughing at and humoring the liberals, suggesting that neither Napoleon as ruler, nor
Junot as military com- mander, intended to grant the liberals’ wishes.

7. B
I: False. See paragraph 5. Portugal’s dependence on Britain is described as military and politi- cal, not economic.
Portugal needed to make terms with the British government to ensure safe passage for its imports of gold from its
colonies. Portugal also depended on England to protect it from France and Spain. No mention is made in the passage
of industrial imports, nor of imports of any kind from England.
II: False. According to the passage, there was no middle class in Portugal (paragraph 5). Secondly, according to the
passage, the lower classes were not demanding political representation; most supported the traditional monarchical
order (paragraph 1).
III: True. The author argues that Portugal’s lack of “progressive agriculture and technological industry” left it
dependent on imports of precious metals from its colonies (paragraph 5). When England took control of the seas,
Portugal made terms and cooperated with the Brit- ish in order to ensure safe passage for these imports (paragraph
5). Portugal’s dependence on England made it a target for Napoleon, who invaded Portugal in order to undermine
England (paragraph 6).

8. C
A: The Inquisition existed before the coming of the Revolution (paragraphs 1 and 4).
B: The passage states that John was regent in 1799 and king between 1816 and 1826 (paragraph 2), but no role by the
Inquisition or Inquisitors in placing him on the throne is mentioned or suggested.
C: Yes. In the first paragraph the author describes how the Inquisition sought to eliminate or “repel” the new ideas
brought to Portugal by the Marquès. The Marquès is portrayed as a rebel against the traditional order, while the
Inquisition protected that order (paragraph 1).
D: This choice is too absolute. The Marquès and his new ideas must have had a fair amount of influence in the 1760s,
if Portugal was “struggling to return to the traditional order” in the 1780s. Furthermore, a “small minority” of
dissenters against tradition existed in Portugal at the time of the French Revolution. These dissenters rejected the past
in favor of new philo- sophical and political ideas (paragraph 3).

9. D
A: This assertion is explained in paragraph 5.
B: The author gives examples of those at the bottom who supported the traditional order, and explains the reasons
for their support (paragraph 1).
C: Examples of members of this minority are given (paragraph 3), and the passage explains why they wanted a
representative government (they were “irked by the despotism of the past,” and wanted freedom of trade, assembly,
press, and thought).
D: Yes. This statement is made in paragraph 2, but it is never explained how the barons acted as models of manners,
and no examples of their behavior in this context are given. The author does mention that the barons were among
those that saw the Church as a “support of private morals,” but this is not an explanation or example of the barons’
own (public) manners.

LOGICAL REASONING (Sectional Test 4) PAGE 4


© Copyright 2020 @LEGALIGHT Education
10. B
A: There is no information in the passage to suggest that the author makes this assumption. The Portuguese monarchy
was vulnerable because of its economic system (paragraph 5), not, as far as we know from the passage, because of
the lack of political representation. The author makes no argument that traditional or unrepresentative systems are
inherently weak or vulnerable.
B: Yes. See paragraph 5. The author states that economic factors led to the political and mili- tary dependence of
Portugal on England and contributed to the invasion of Portugal by France and Spain. The author also indicates that
Spain’s economic dependence on imports from its colonies led to its (failed) attempt to fight Britain and thus to its
eventual alliance with and dependency on France.
C: While this is an assumption made by the author (paragraph 5), it is not directly relevant to or supported by the new
information given in the question. There is no indication in the passage or the question stem that France lacked a
strong middle class, or that it lacked industrialization and was dependent on colonial resources.
D: The passage never suggests that the Portuguese monarchy levied high taxes or that it built up its naval forces.

11. B
Here is an overview of the shifting alliances described in the passage. At first Spain, Portugal, and England all waged
war on France in 1793 (paragraph 4). Then, Spain negotiated a separate peace with France (paragraph 4), and the two
eventually joined forces (paragraph 5). France refused to negotiate a separate peace with Portugal (paragraph 4), who
eventually joined forces with England in order to protect itself against Spain and France (paragraph 5).
A: While the Portuguese royal family fled to Brazil (paragraph 6), no military alliance is sug- gested.
B: Yes. Spain joined with France in invading Portugal (paragraph 5). In 1793, Portugal “fol- lowed Spain in a holy war
against France” (paragraph 4).
C: No alliance between Spain and Brazil is mentioned.
D: The passage suggests that Charles III and Louis XV had some similar ideas (paragraph 1), but no alliance, military or
otherwise, is mentioned.

12. D
A: While the author suggests that economic dependency may in some cases lead or contribute to political dependency
(as in the case of Portugal), he says nothing about the reverse, i.e., that political dependency may lead to economic
dependency.
B: It was Portugal’s reliance on imports of gold and other metals from its colonies (paragraph 5) that impeded the
development of both its agricultural and industrial sectors.
C: There was a minority in Portugal who hoped for greater representation and liberty (paragraph 3), but they are not
described as “well-organized” (the fact that “liberal leaders” existed is not enough to support this inference). Secondly,
while they did “flock” to Napoleon after the fact (paragraph 6), the passage does not indicate that this “timid minority”
(paragraph 3) actually played a role in, or paved the way for, Napoleon’s invasion.
D: Yes. If the “small minority” described in paragraph 3 “dreamed of . . . free trade,” then free trade must not have
existed under the Portuguese monarchy.

Passage 3
13. B
A: No. Paragraph 4 states that diabetes mellitus is characterized by glucosuria, while diabetes insipidus is not. The
scenario in the question would contradict, not support this distinction.
B: Yes. The passage states in paragraph 4 that diabetes insipidus, unlike mellitus, is not charac- terized by glucosuria.
The finding in the question would contradict this distinction.
C: No. The passage only indicates that diabetes mellitus types I and II may in the future be found to be unrelated
(paragraph 6). Also, glucosuria in a patient with insipidus is irrelevant to the distinction between mellitus types I and
II.
D: No. Paragraph 4 indicates that insipidus and mellitus are caused by malfunctions in different organs or parts of the
body. Their effects on different body parts are never discussed.

14. D
A: No. According to the author, diabetes represents at least two distinct conditions, insipidus and mellitus.

LOGICAL REASONING (Sectional Test 4) PAGE 5


© Copyright 2020 @LEGALIGHT Education
B: No. While diabetes is no longer seen as a single disease, the author does not indicate that any form of it has been
disqualified as diabetes.
C: No. The only characteristic in common that is mentioned in the passage is polyuria (para- graphs 1 and 4).
D: Yes. Diabetes insipidus and mellitus I and II are all characterized by polyuria, but only mel- litus I and II cause
glucosuria. Type I is also distinct from type II in that those with type I must receive insulin.

15. B
A: No. Different forms of diabetes originate from different parts of the body. If this choice were correct, it would have
been discovered that diabetes mellitus originates from the pituitary gland instead of from the pancreas (paragraph 4).
B: Yes. “Diabetes” or excessive urination was once thought to be a disease or condition in and of itself. Now excessive
urination is seen as a symptom of two other diseases, diabetes in- sipidus and diabetes mellitus, the second of which
is divided into types I and II. Only type I requires treatment with insulin (paragraph 5).
C: No. Diabetes mellitus type I and type II (two of the three types or conditions identified) are related, both arising, as
far as we know, from a dysfunctional pancreas. The author states that we may discover in the future that they are in
fact unrelated (paragraph 6), but no such evi- dence, based on the passage, exists to date.
D: No. A distinction between physiological and mental disorders is not discussed in the passage, nor is there any such
contrast drawn.

16. C
I: True. If the need for insulin depended on diet, it would weaken the belief that there are two types of diabetes
mellitus. The difference would likely lie in aspects of the patient’s lifestyle, not in a difference between two types of
disease (see paragraph 5).
II: True. In the current conception of diabetes, diabetes insipidus does not result in glucosuria (paragraph 4).
III: False. The current conception of the disease already includes a role played by the pancreas in diabetes mellitus
(paragraph 4). Thus the statement would in no way indicate that the current conception is mistaken.

Passage 4
17. C
A: No. In the first paragraph, the author argues that value is nonexistent when no alternatives are available.
B: No. Actions are virtuous when they achieve values (paragraph 1). Value is defined by the author as one’s own
happiness, not the happiness or good of the community or others within the community (paragraph 2). This choice
contradicts the author’s ethic of selfishness as it is expressed throughout the passage.
C: Yes. In the opening paragraph, the author claims that “‘value’ presupposes…the necessity of action in the face of an
alternative.” Virtue is defined as the actions one takes in order to achieve values. If you have no choice but to pursue
a given course of action, your actions are not virtuous.
D: No. This statement directly contradicts the author’s repudiation of a morality based on self- sacrifice (paragraphs 2
and 7).

18. D
A: No. This claim is supported in paragraph 5, but it is too narrow to be the main point of the entire passage.
B: No. Happiness for whom? This choice is too vague to capture the author’s advocacy of an ethic of selfishness, and
fails to include the passage’s critique of self-sacrifice.
C: No. The author rejects the “eternal, absolute, objective” moral standards of those who define value as the good of
others (paragraph 6). However, she does not (either specifically or in the passage as a whole) argue against the
existence of any eternal or objective standards.
D: Yes. The author presents her arguments for selfishness in the first half of the passage, and then moves on to a
related critique of moralities based on self-sacrifice for the good of others.

19. C
A: No. The author rejects moralities based on need (paragraph 7).
B: No. This choice is inconsistent with the theme of the passage, which is that morality is entirely defined by the
achievement of one’s own happiness (paragraph 2). The author does not advo- cate treating others badly, but neither
does she accept the idea that morality is to be defined by reference to anyone but oneself (paragraph 4).

LOGICAL REASONING (Sectional Test 4) PAGE 6


© Copyright 2020 @LEGALIGHT Education
C: Yes. The passage portrays dependency on others in a strongly negative light (paragraph 7). The author clearly would
support the theme of self-reliance embodied in this motto.
D: No. This is inconsistent with the author’s critique in paragraph 5 of those who use intention as a “standard of virtue.”

20. D
A: No. The author’s own express purpose is to evaluate and reject moral codes (those based on self- sacrifice) held by
others.
B: No. The author does reject intention as a standard of virtue (paragraph 5), but this is not the meaning of the cited
phrase in the context of the second paragraph. This answer choice is the right answer to the wrong question.
C: No. This choice contradicts the main idea of the passage, which holds that individuals should not sacrifice themselves
for the good of the whole. Beware of choices that take words (“end”) out of context.
D: Yes. In this part of the essay (the second paragraph) the author argues that every individual exists only for himself,
and should act only in order to secure his own happiness (“the high- est moral purpose”) rather than the happiness of
others.

21. A
A: Yes. The author claims that those who sacrifice for others become in turn dependent, “in the position of a parasite
whose needs must be filled by others” (paragraph 7). By depicting the “disgraceful role” of such a “beggar,” the author
gives evidence that she would support the elimination of any system or policy whose mission was to fulfill the needs
of those that do not or cannot support themselves.
B: No. The author places high value on self-reliance and self-sufficiency (paragraph 7). Socialized medicine would
contravene, not embody, those principles as she defines them.
C: No. Conscription into the military entails forcing the individual to (potentially) sacrifice his own life to protect the
nation. This contradicts the central thrust of the passage.
D: No. This choice is similar to choice B. The author would reject a policy that provided a guar- anteed benefit or
standard of living, turning citizens into (in the mind of the author) “beggars” and “parasites.”

22. C
A: No. This choice is only partially correct (and therefore wholly incorrect). The author calls on us to prioritize our own
desires and needs, not to balance them against the needs of others.
B: No. This choice is too vague. Self-esteem is maximized when we act in accordance with the specific code of value
set out in the passage, as the author suggests in the final paragraph (paragraph 7).
C: Yes. Self-sacrifice destroys self-esteem (paragraph 7); maintaining independence of mind would have the opposite
effect (paragraph 4).
D: No. The author criticizes those who use their intentions as justification for their actions (para- graph 5).

23. D
A: No. The author uses similar ironic wording throughout the passage (see for example paragraph 7). By referring to
morality that negates morality, the author is suggesting how wrong and destructive she believes a morality of self-
sacrifice to be, rather than indicating an internal contradiction within the concepts themselves. She herself puts
forward a moral code based on a reasonably clear and consistent definition of value.
B: No. The author advocates such standards (paragraph 4).
C: No. The author rejects a certain moral code, not the ideas of value and morality themselves. D: Yes. By claiming that
a morality that requires self-sacrifice is a negation of morality, etc., the author underscores the absolute and
fundamental nature of her critique. She also
highlights the contrast between this supposedly debased code of false values and her own “highest moral purpose”
based in selfishness.

Passage 5
24. C
A: No. The author claims that society’s unfriendliness helps make transcendental experiences pos- sible (paragraph 2),
not that it develops from transcendental movement.
B: No. This contradicts the passage, which claims that society’s unfriendliness is a positive factor that contributes to
transcendental experiences (paragraph 2).

LOGICAL REASONING (Sectional Test 4) PAGE 7


© Copyright 2020 @LEGALIGHT Education
C: Yes. This is the main idea of the second paragraph.
D: No. In the second paragraph, the author clearly believes alienation from society to be a posi- tive influence on
artistic inspiration.

25. A
A: Yes. Paragraph 4 describes how the author’s original conception is only a starting point. The finished product can
be neither predicted nor anticipated. In paragraph 5, the author says that each painting is an “unexpected and
unprecedented resolution.”
B: No. The passage states that archaic societies were more practical (paragraph 6), but never claims that artistic
inspiration is impractical.
C: No. While the end product may be unpredictable, it does represent “resolution of an eternally fa- miliar need”
(paragraph 5). Thus it reflects the needsnad desires of all people, including the artist.
D: No. This is a misreading of paragraph 4. The author is expressing the unpredictable nature of art, not its
otherworldliness or eeriness. (Be careful not to take words out of context.)

26. C
A: No. According to the author, the “miraculous” nature of art is its ability to provide an unex- pected revelation to all
who experience it (paragraph 5). Gods and monsters are discussed later in paragraphs 6 and 7, in a different context.
B: No. The representation of reality in other forms is discussed later in the passage (paragraph 6). The “miraculous”
nature of art relates to a different issue—the unpredictability of the final product.
C: Yes. The author describes art as “miraculous” because at the moment of completion, it be- comes an entity
independent of the intentions and original conceptions of the artist (para- graph 5).
D: No. The picture conveys “eternally familiar” needs and desires, not the specific desires the art- ist may have
intended to express (paragraph 5).

27. B
A: No. According to the author, art does not express the artist’s individual perspective (paragraph 5).
B: Yes. The author uses the analogy to describe the independence of the finished work from the intentions of the artist
(paragraph 3). The components of the painting move and interact by their own volition, as do actors on a stage.
C: No. Contemplation of the world is never discussed in this context.
D: No. The analogy between stage and frame is neither made nor implied by the author.

28. B
A: No. The author does not suggest that modern art is less able to express emotion.
B: Yes. In paragraph 7, the author describes the loss of belief in gods and monsters as a source of frustration and
melancholy.
C: No. The value of art’s role in society is never discussed.
D: No. This choice takes the reference to “finite associations” in paragraph 6 out of context. It is not these finite
relationships themselves that lead to melancholy, but the unavailability of gods and monsters to express and portray
transcendence.

29. B
I: False. The correct choice(s) will NOT undermine the author’s contrast between archaic and modern art and society.
This statement would attack the author’s claim that finite associations are increasingly made in modern society
(paragraph 6).
II: False. The author claims in paragraph 6 that the archaic artist lived in a more practical society than ours. Thus this
statement would weaken the author’s claims.
III: True. This statement would have no effect on the author’s claims. The depiction of monsters and gods is no longer
an essential means of artistic expression (paragraph 7). If such depic- tions are often poorly done, it would have no
relevance to the author’s points about the difference between archaic and modern art.

30. B
A: No. This statement can be inferred from paragraph 5. The correct answer will be the statement that is least
supported by the passage.

LOGICAL REASONING (Sectional Test 4) PAGE 8


© Copyright 2020 @LEGALIGHT Education
B: Yes. According to the author, modern art’s most profound moments express the frustration arising from our lack of
belief in gods and monsters (paragraph 7). Archaic art utilized gods and monsters (paragraph 6), but the author does
not use their presence or absence as a stan- dard of profundity.
C: No. This is directly stated by the author in paragraph 7.
D: No. This is directly supported by the author’s discussion of transcendence and separation from society in paragraph
2.

Passage 6
30. A
A: Yes. In paragraph 8, the passage states that Poe “neatly summed up the situation” in that quote. The situation
referred to is in the previous paragraph, in which a groan in reaction to a good pun may indicate “a hidden envy.”
B: No. Neither Poe nor the author depicts puns as a sophisticated form of humor. C: No. Poe’s statement relates only
to puns, not to other forms of humor.
D: No. This choice is too extreme. Poe does not imply that punning is funnier than other forms of humor.

31. C
A: No. Johnson is condemning punning itself, not singling out particularly playful puns for criti- cism.
B: No. This choice is too extreme. While punning may, according to Johnson, involve the misuse of the tools with which
intelligence is expressed (i.e., words), we cannot infer from this that he believes that punners are themselves
unintelligent.
C: Yes. Johnson describes punning as “trifling” with words, and compares it to grand larceny (invading the national till,
in paragraph 4). Thus he sees it as both frivolous and illicit.
D: No. This choice contradicts Johnson’s meaning. He views punning as undermining, not as embodying, intelligent
expression.

32. B
A: No. People’s reaction to sexual humor has no relevance to the punster’s courage (or lack thereof).
B: Yes. This fact would indicate that punning is not particularly risky, and so undermine the author’s claim in paragraph
6 that punning requires bravery.
C: No. This would strengthen, not weaken, the author’s argument that punsters risk failure and public humiliation
(paragraph 6).
D: No. This choice is too weak to challenge the passage. The author does not argue that all puns fail to be entertaining
(he or she writes “Punning can be highly entertaining” in paragraph 6). Thus the success of some puns has no effect
on the passage’s claims.

33. C
A: No. The correct answer will indicate that puns sit on top of a hierarchy of humor. This choice is part of the author’s
definition of a pun (paragraph 1), but does not compare puns to other forms of wit.
B: No. This statement suggests that puns may be skillful, but not that they are more skillful than other forms of humor.
C: Yes. This quote (paragraph 5) indicates that punning arises from the greatest form of genius, and so would lend the
most support to the claim that punning is a superior form of humor.
D: No. This statement describes what punsters do (paragraph 6), but not their great skill at doing it compared to other
humorists.

34. D
A: No. Slander is telling a destructive lie about another person. The author has a positive, not a negative attitude
towards punning.
B: No. The author thinks puns can be skillful and entertaining, not destructive.
C: No. The author says that good punning can be entertaining, but not that all puns entertain (paragraph 6).
D: Yes. The inveterate punster is brave, and punning is “always a risky business” (paragraph 6).
Thus punning fundamentally involves courage.

LOGICAL REASONING (Sectional Test 4) PAGE 9


© Copyright 2020 @LEGALIGHT Education

Passage 7
35. B
A: No. This choice is inconsistent with the passage. The author believes that good fiction deals with the senses and
concrete details, not philosophical abstractions (paragraph 1).
B: Yes. The author writes that human knowledge, and so fiction, begins with the senses, and that the way to appeal to
the senses is through the use of concrete detail (paragraph 1).
C: No. Successful fiction portrays the details and mysteries of life. The author never discusses literature that explains
the process of understanding.
D: No. The author argues that fiction should go beyond simply presenting a “slice of life” (para- graph 5).

36. B
I: True. In paragraph 3, the passage indicates that many readers believe that defining or as- signing meaning to a
symbol equates with understanding the story.
II: True. Many readers are put off by symbols because they see them as something that only an elite group, of which
they are not a member, can comprehend (paragraph 3).
III: False. The passage never refers to symbols with sociological meaning. Sociology is mentioned in the first paragraph
as a concern of beginning writers, but is never connected to the use or appreciation of symbols.

37. A
A: Yes. The author argues that a successful author must address the details and mystery of life as well (paragraph 2).
B: No. The author directly states that readers who do not have educated minds may understand good fiction
(paragraph 5).
C: No. In paragraph 5, the author criticizes those who believe that identifying the theme is the key to understanding.
The author believes that it is the details that make for good fiction (paragraph 1).
D: No. This choice is too extreme. Psychological insight and sociological themes do not alone cre- ate good fiction
(beginning of paragraph 2), but the passage never claims that psychology and sociology should be excluded altogether
from literature.

38. A
A: Yes. “Mystery” is contrasted with reality and the “canny” or knowable aspects of life. Thus mystery in this context
indicates the enigmatic or hidden elements of existence.
B: No. “Mystery” in the context of the passage refers to a quality of life that is communicated by good literature, not
to difficulties in understanding literature itself.
C: No. Be careful not to take words out of context of the passage. Nowhere does the author refer to the genre of
mystery or suspense novels, or to the use of suspense as a technique.
D: No. The author uses the word “mystery” to describe qualities of good writing, not problems encountered by writers
or readers.

39. C
A: No. The author states that people are more comfortable expressing abstract ideas than offer- ing concrete
descriptions (paragraph 1). The passage never makes any comparison between people’s comfort level expressing ideas
in general versus expressing feelings. Thus the answer choice is not relevant to the passage and does not weaken the
author’s claim.
B: No. The author makes the claim about the world of the fiction writer in order to argue that good fiction concerns
itself with the details or “dust” of life. The unpopularity of theoretical texts has no direct relevance to that issue.
C: Yes. In the first paragraph, the author writes, “[The fiction writer] appeals through the senses, and you cannot
appeal to the senses with abstractions.” Thus good writers concern themselves with “matter” or concrete details. If
writers were able to engage the senses through abstraction, it would weaken the author’s contention.
D: No. The comparative significance of emotional writing for writer and reader has no direct relevance to the author’s
contention about the use of concrete detail.

40. A
A: Yes. The final sentences of the passage suggest that if everyday life were all we needed, lit- erature would not exist.
Therefore, literature must include or provide something beyond a depiction of everyday life.

LOGICAL REASONING (Sectional Test 4) PAGE 10


© Copyright 2020 @LEGALIGHT Education
B: No. While this makes common sense, it is never an issue in the passage.
C: No. The foundation of human knowledge is in the senses, and so good literature appeals through those senses
(paragraph 1). However, literature itself is not described as the founda- tion of human knowledge.
D: No. Engagement of the senses is a quality of, not a motivation for, the writing of good litera- ture. Also, literature
as an additional way to engage the senses is never discussed.

41. D
A: No. The author does say that fiction which is only based on emotion and perception is dull (beginning of paragraph
2). However, this is not the same distinction as that between ideas and people, and the author does not use his or her
own enjoyment as a standard.
B: No. All writers should avoid abstraction, not just beginning writers (middle of paragraph 2).
Level of skill is not the issue.
C: No. The author does not draw a distinction between ideas and mystery, nor does the author imply that people have
difficulty in understanding mystery.
D: Yes. This is directly supported by the middle of paragraph 1, and by the discussion of “dull- ness” in the beginning
of paragraph 2.

Passage 8
42. B
A: No. The Great Depression in one of a series of events described in the passage. Also, the pas- sage does not follow
chronological order (see the first sentences of paragraphs 1 and 2).
B: Yes. The first paragraph sets out the main theme of shifts in political allegiance. Each fol- lowing paragraph discusses
how various groups came to reject unregulated capitalism and the effects those defections had on political power and
policy.
C: No. This choice is too narrow; this is just one of the points made by the author (middle of paragraph 2).
D: No. How unionization progressed is discussed in the third paragraph only; the passage as a whole is not a history of
unionization.

43. A
A: Yes. 1929 was a “watershed year,” in which the crash started a profound political shift from Republican to
Democratic power (beginning of paragraph 1).
B: No. Marxist ideas are never mentioned.
C: No. Both the political shift and the labor movement were effects of the crash and subsequent economic depression.
The depression changed the thinking of workers, which then resulted in both increased unionization and political
realignment.
D: No. The 1921 depression “had little effect on national politics” (paragraph 2).

44. C
A: No. The correct answer must be based on the passage, not common sense or outside knowl- edge. Farming
equipment is never mentioned.
B: No. The cost of seed and grain is never discussed.
C: Yes. In 1921, prices for farm products fell but the farmers’ “costs, chiefly mortgage pay- ments to banks, remained
high” (beginning of paragraph 1).
D: No. Acquisition of manufactured goods is never discussed.

45. D
A: No. Wages were increasing slightly between 1921 and 1929, but no related increase in union- ization is described
in the passage. No figures on wage levels during the post-1929 unioniza- tion drive are given.
B: No. Young intellectuals are described as calling for “governmental regulation and oversight” (paragraph 3). No direct
connection is drawn by the author between their activities and union- ization.
C: No. Rural poverty (paragraph 2) is not given by the author as a causal factor.
D: Yes. The sharp increase in unemployment following the crash encouraged the rejection of unregulated capitalism
and facilitated the organization of labor into unions (beginning of paragraph 3).

LOGICAL REASONING (Sectional Test 4) PAGE 11


© Copyright 2020 @LEGALIGHT Education

46. A
A: Yes. The passage describes how economic changes (the crash of 1929 and the ensuing de- pression) contributed to
a change in attitudes and beliefs about the legitimacy of organized labor and of the old political order (paragraph 3).
This led to the increase in political weight and power of farmers and laborers (end of paragraph 3) and to the passage
of new kinds of legislation (beginning of paragraph 4). Similarly, in this answer choice, an economic change leads to a
change in attitudes, which contributes to political and legislative changes.
B: No. In the passage, economic change causes a change in attitudes or beliefs. In this choice, be- liefs and attitudes
come first. Furthermore, although the passage states that not all of the New Deal legislation survived (paragraph 4),
most of it did; to characterize the Supreme Court’s actions as a “backlash” would be too extreme.
C: No. The passage describes changes in beliefs and attitudes as playing a fundamental role (be- ginning and middle of
paragraph 3). No such factor exists in this answer choice.
D: No. As in choice C, this answer lacks any discussion of changes in attitudes. Furthermore, there is no international
factor at play in the passage.

Passage 9
47. D
A: No. The passage does not tell us who conceived of the term “health food.” The author does assert that the term is
artificial and false; he or she would not agree that such products are “useful” (paragraph 1).
B: No. The term, according to the author, falsely suggests that “health foods” are healthier than non-“health foods”
(paragraph 1).
C: No. The passage asserts that the term cannot be defined (paragraphs 1 and 3). The statement in this answer choice
(meeting all the dietary needs of most people) is suggested as one of the false or misleading definitions that people
use (paragraph 1).
D: Yes. The term is labeled “false and misleading” in the middle of paragraph 1.

48. C
A: No. The author completely rejects the term “health food” as misleading and impossible to define.
B: No. Human milk is not described as being more deficient in vitamins than any other food. In fact, it is described as
the most nutritionally complete food (end of paragraph 1).
C: Yes. The passage claims that human milk (for babies) is the only food that by itself supplies all necessary nutrients
(paragraph 1).
D: No. Sun-drying of milk is never discussed. The correct answer must be based on information given in the passage,
not on common sense or outside knowledge.

49. B
A: No. The author never accepts that the health food movement may in fact have some connec- tion to good nutrition.
B: Yes. The author makes this statement in the context of explaining why sick people are espe- cially vulnerable to the
false claims made by purveyors of “health food” (paragraph 6).
C: No. While the author states that food is related to health, he does not go so far as to claim that nutrition can be
used to cure disease.
D: No. The author never criticizes the medical establishment; if anything he presents it in a posi- tive light, in opposition
to the health food movement (paragraphs 3 and 5).

50. B
A: No. A belief that something is true, even a widespread belief, does not constitute valid evi- dence that it is in fact
true.
B: Yes. The author claims that few of the health food movement’s claims have been tested, and that none can be
substantiated (paragraph 4). If valid tests have been done showing that
a particular type of food does in fact promote health, it would undermine this part of the author’s argument. It would
also suggest that there may in fact be, in the future, some way of defining “health foods.”
C: No. The author accepts that food and nutrition is generally related to health (paragraph 6).

LOGICAL REASONING (Sectional Test 4) PAGE 12


© Copyright 2020 @LEGALIGHT Education
D: No. This is not inconsistent with the author’s argument. The passage never claims that the extract was harmful in
and of itself, only that it did not have beneficial effects (paragraph 5), and that it caused people to forgo potentially
beneficial treatments.

51. B
A: No. The question stem does not indicate that olives, honey, and dates are among the nutritious ancient foods.
B: Yes. The author argues that traditional foods are not necessarily more nutritious (paragraph 2), but not that all
traditional foods are non-nutritious. Therefore, the new information in the question stem would have no impact on
the author’s argument.
C: No. This choice is partially correct, and therefore wrong. While the first half is supported by the passage, the second
half is not. The author discusses traditional foods as one way of argu- ing that “health food” cannot be defined
(paragraphs 1 and 3).
D: No. The fact that nutritious foods were eaten in the past has no direct relevance to nutrition as a science.
Furthermore, the author’s statement that nutrition is a legitimate science (paragraph
6) is not part of his argument against the possibility of defining “health food.”

Passage 10
52. D
A: No. This choice is too extreme. The passage indicates that if an explanation meets the two requirements, it can be
accepted as a true explanation.
B: No. The fourth paragraph discusses ways in which an explanation can be tested.
C: No. The passage says that discovery of new laws is not a requirement (paragraph 8) and the author does not indicate
how often explanations might lead to new laws. Therefore, “often” is too extreme.
D: Yes. The author states that explanatory relevance is not enough, as “it does not explain why” a phenomenon occurs
(paragraph 3). The passage then goes on to discuss the second neces- sary condition, testability, and the example of
the Torricelli barometer.

53. A
A: Yes. The barometer is offered as an example of testability, and we can infer from paragraph 3 that an explanation
of why a phenomenon occurs is part of empirical testing. Finally, the description of the results of testing the barometer
focuses on reasons why the column falls as altitude increases (paragraph 5).
B: No. The author indicates that the phenomenon passes that test (paragraph 6). C: No. The mercury falls as altitude
increases (paragraph 5).
D: No. The author states that the explanation provided for the rainbow does meet the require- ment for explanatory
relevance (paragraph 2).

54. C
A: No. The explanandum phenomenon is the phenomenon itself (paragraph 7). The explanan- dum sentence simply
describes the phenomenon. The necessary ingredients of an explanation are explanatory relevance and testability
(paragraph 1).
B: No. The explanandum phenomenon is the event to be explained (paragraph 7).
C: Yes. These are the two requirements for scientific explanations (paragraph 1).
D: No. The passage says that science attempts to create “a conception of the world that has a clear logical bearing on
our experience” (paragraph 1). However, “clear logic” is never given as a criterion for scientific explanations
themselves.

55. A
I: True. The explanation of rainbows given in the passage is that they are caused by an interaction of light and water
(paragraph 2). Water plays no role in a rainbow formed by a prism.
II: False. Just because a day is dry doesn’t mean no water is present (from a water hose, for example).
III: False. The passage claims that rainbows are formed by light and water (paragraph 2). This choice is entirely
consistent with the explanation given.

LOGICAL REASONING (Sectional Test 4) PAGE 13


© Copyright 2020 @LEGALIGHT Education
56. B
A: No. Explanatory relevance is achieved when good grounds are given for believing that the phenom- enon could
occur. The failure of the rainbow to actually appear relates to testability, not relevance.
B: Yes. The second condition for acceptable explanations is testability. If the rainbow never ap- pears despite the
illumination of water in the way set out in the passage (paragraph 2), the explanation fails.
C: No. The explanation would fail the second requirement: testability (paragraph 4).
D: No. This choice is too extreme. The explanation fails to meet the second of the two criteria for proving its truth.
However, the passage never says that such failure constitutes proof of its falsehood.

Passage 11
57. C
A: No. This choice is too narrow. The author does refute Sullivan on this basis (paragraph 3), but this is only one point
made within the passage, not the main point of the entire passage.
B: No. This choice is too broad. The passage specifically explores how schizophrenia may relate
to and reflect modernity, not how societal characteristics are often reflected in the thought pro- cesses of the mentally
ill.
C: Yes. Throughout the passage the author considers qualities of modernism and the peculiar characteristics of
schizophrenic thought, and speculates at the end about what each may indicate about the other.
D: No. This choice misses the issue of the passage. The author concerns himself not with biologi- cal or environmental
causes of illness, but with similarities and affinities between schizophren- ic and modernist thought.

58. D
A: No. It was Aragon, not Duchamp, who employed radical reversals (paragraph 1). Furthermore, the author rejects
Sullivan’s description of schizophrenic thought as regressive or childlike (paragraph 3). He states that schizophrenic
cognition is in many ways similar to Duchamp’s perspective (paragraph 2), thus Duchamp’s imagery would not be
characterized as regressive by the author.
B: No. Duchamp does not illustrate standard beliefs, but instead contemptuously rejects them (paragraph 1).
C: No. This choice takes words from the passage out of context. First there is no discussion of subcategories in the text
provided. Secondly, Duchamp’s category includes not actual “physi- cal phenomena,” but rather skewed perspectives
on physical realities.
D: Yes. See paragraphs 1 and 2.

59. B
A: No. The author never suggests that more people suffer from the disease than in the past, nor does he propose
complexity or disorder of the world as an actual cause of schizophrenia. This would be a misinterpretation and overly
literal reading of his discussion of the affinities be- tween schizophrenic and modernist thought.
B: Yes. Through his discussion of the modern awareness of multiple points of view (paragraph
4) and of how parallels between schizophrenia and modernism may indicate something about the modern sensibility
(paragraph 5), the author indicates that the modern world and sensibility have become more complex and alienated
than in the past.
C: No. The wording in this choice is too extreme. The author poses the question of whether this might be the case
(paragraph 5), but never definitively answers the question. This paragraph tells us that the author is considering
various interpretations, not proposing clear and defini- tive answers.
D: No. This choice takes the author’s reference to “homelessness of mind” (paragraph 3) too literally. The passage
never suggests practical social conditions or problems as causal factors in schizophrenia.

60. C
A: No. Representation of “the object in the subject” (the object as perceived by thetp) oe arises from the poet’s self-
reflection and self-consciousness. “Object” in this context does not mean objective or purpose; the phrase refers to
what the poet does, not Schiller’s purpose in depicting the poet.
B: No. This choice takes the words “object” and “subject” out of context. The object is not a subcategory within a larger
category or subject. Rather, the object is the thing perceived and represented by the subject or poet.

LOGICAL REASONING (Sectional Test 4) PAGE 14


© Copyright 2020 @LEGALIGHT Education
C: Yes. According to the passage, the poet’s self-consciousness (middle of paragraph 4) creates awareness of the
multiple perspectives or ways of seeing things existing within himself (“self-divided”). This awareness leads him
describe the character of things as something filtered or mediated through his own perceptions or point of view.
D: No. Self-awareness leads to the modernist’s presentation of the object in the subject (the object as the poet
perceives it—see paragraph 4). Thus the poet’s (and in general the modernist’s) work is characterized by self-
awareness; the passage never indicates that his work focuses on self-awareness as a topic or “subject.”

61. A
A: Yes. The author refutes Sullivan’s claim that schizophrenia represents a regressed or develop- mentally primitive
state (paragraph 3), and argues that it suggests an alienated (isolated from
oretality), n childlike, condition. If it were to be shown that the disease in fact corresponded
to a form of arrested development, it would undermine the author’s position.
B: No. “Primitive” in the context of the passage means a regressed or childlike personality (beginning of paragraph 3).
The author never argues that schizophrenia is unique to advanced or modern societies. Therefore this finding would
not undermine the author’s position.
C: No. This claim is entirely consistent with the author’s depiction of the schizophrenic mind as “homeless,” slipping
from one curious perspective to another (paragraphs 2 and 3).
D: No. The author describes the affinities or similarities he sees between modern culture and schizophrenia, but does
not claim that modern culture or environment causes the illness. Thus the existence of biological causes or
preconditions would not be inconsistent with his position on Sullivan’s work.

62. D
A: No. Based on the context of the passage, and specifically on the author’s point of disagree- ment with Sullivan
(paragraphs 3 and 6), “unintelligible” suggests our inability to understand or share in the schizophrenic’s peculiar
perspective on reality (see example in paragraph 2). “Unintelligent” corresponds to Sullivan’s claim that schizophrenic
thought arises from fail- ure or lack of “higher referential processes” (childlike, in the author’s terms). While “foreign”
could correspond to strangeness or unintelligibility, “unconventional” does not correspond to Sullivan’s view of
schizophrenic thought. This choice is only half right, and so is incorrect.
B: No. See discussion for A. “Unnatural” does not correspond to the author’s use of “unintelli- gible,” not does
“unknown” connect to Sullivan’s “primitivity hypothesis” (paragraph 3).
C: No. See discussion for A. The author describes how multiplicity (and oddness) of perspective character- izes
schizophrenic thought (beginning of paragraph 2). Thus “multiperspectival” would correspond to “unintelligible” (the
author’s view), not “unintelligent” (Sullivan). “Singular” corresponds to neither.
D: Yes. See discussion for A. Something that is illegible cannot easily be understood, and yet may have value or interest
to us if we could in fact decipher it. This is similar to the author’s consideration of what schizophrenia may be able to
tell us about the modern mind and condition (beginning of paragraph 5). Something that is illogical lacks rationality—
it is de- fined negatively by what it is missing, i.e., reason. This corresponds to Sullivan’s depiction of schizophrenics as
regressed and childlike, a characterization that the author rejects in the beginning of paragraph 6, and earlier at the
end of paragraph 3.

63. B
A: No. The author would in fact disagree, but the rest of the choice is too extreme. Schizophrenic thought has many
similarities to modern modes of thought and belief (paragraphs 2, 5, and 6). However, “perfectly reproduce” goes too
far to be supported by the passage.
B: Yes. The primary purpose of the passage is to explore and consider the significance of simi- larities between the
two. See in particular the last two paragraphs.
C: No. The author would disagree. The statement in the question contradicts the primary pur- pose of the passage,
which is to point out similarities and consider their significance.
D: No. This choice is only partially correct (and so quite wrong). Some do make the claim that thought processes similar
to those of schizophrenics are necessary for creativity (paragraph 1), but the author himself never supports this claim.

LOGICAL REASONING (Sectional Test 4) PAGE 15


© Copyright 2020 @LEGALIGHT Education

Passage 12
64. A
A: Yes. The correct answer will be the choice that either contradicts the passage or is not men- tioned as a gesture of
dominance. According to the author, an inhibited bite is a submissive gesture (paragraph 2), thus this answer choice
contradicts the passage.
B: No. The author describes a dominant wolf staring at another animal in paragraph 1. C: No. Riding up is listed as a
way of emphasizing dominance (paragraph 4).
D: No. Hip slams are described as a means of displaying dominance (paragraph 4).

65. D
A: No. The author does not claim that submission always eliminates violence and maintains har- mony, just that it is
necessary for the maintenance of harmony.
B: No. The author himself states that the study of vocalizations may add to our understanding of the wolves’ body
movements (paragraph 6).
C: No. The author argues that submission serves to maintain group harmony (paragraph 5).
Therefore, if a wolf who shifts from submissive to dominant behavior disrupts that harmony, it would be consistent
with the author’s interpretation.
D: Yes. Schenkel’s work was based on wolves in captivity (paragraph 3). The author also states that the expressions he
and others describe are rarely observable in the wild (paragraph 3). If the behavior described in the passage is
significantly different than behavior in the wild (as indicated by this answer choice) it would cast doubt on the validity
of the author’s data. If his evidence is flawed, this would undermine his interpretation of wolf behavior.

66. A
A: Yes. The author explains how wolf gestures contribute to group harmony (paragraph 5), and that the language of
wolves is not simplistic (paragraph 6).
B: No. According to the author, this would be an incorrect interpretation of certain wolf gestures such as rolling over
(paragraph 5).
C: No. Acts such as rolling over in submission would be degrading for a human, but are not so for a wolf (paragraph 5).
D: No. This choice is too strong. The author never indicates that communication between wolves is either rigid
(unchanging) or infallible (that is, that wolves never miscommunicate).

67. A
A: Yes. Based on the information in the passage, we know from the scenario proposed by the question that B is
dominant over the muzzle-licking C (see end of paragraph 1), and that A and C’s stiff-legged approach indicates their
dominance over D (paragraph 4). However, the relationships between A on one hand, and both C and B on the other,
are still not defined. If A hip slams B, a sign of dominance (paragraph 4), we now know that A dominates B, who
dominates C, who dominates D.
B: No. We already know C to be dominant over D.
C: No. If B hip slams (dominates) A, the pecking order between A and C is still undefined. D: No. If A hip slams
(dominates) C, the hierarchy between A and B is still unclear.

68. D
A: No. The passage never indicates that mingling with the wolves inside their cage will provide more accurate
observations than Schenkel’s post outside the cage did.
B: No. Gender is never discussed by the author in connection to dominance relationships. Thus, there is no reason to
infer that non-vocal expressions could be better understood by taking gender into account.
C: No. The author makes no direct connection between the frequency of an expression and its meaning.
D: Yes. In the final paragraph, the author predicts that improved understanding of the lan- guage of wolves will come
from relating gesture and movement to vocalization.

69. B
A: No. Retracting the corners of the mouth is a sign of passive submission (paragraph 1), just the opposite of eagerness
or willingness to fight.

LOGICAL REASONING (Sectional Test 4) PAGE 16


© Copyright 2020 @LEGALIGHT Education
B: Yes. In the second half of paragraph 1, this is described as a “submissive grin.” Thus the wolf who is not grinning
would be the dominant animal.
C: No. This is a sign of submission (paragraph 1). It is unlikely that the submissive animal is preparing to fight; the
author describes submissive behavior as a way of maintaining group harmony (paragraph 5).
D: No. The author does not indicate that submissive and dominant behaviors occur only among family members.

Passage 13
70. C
A: No. Meaning found by readers in a story is never discussed. The passage is about how authors create stories and
find meaning in their own creations.
B: No. Meaning rises up behind the story, in retrospect (paragraph 1). It is not available before the fact as a guide to
the story.
C: Yes. According to the author, meaning arises in retrospect or hindsight, emerging for the author only after the
writing is done (paragraph 1).
D: No. Meaning comes in part out of the recognition of recurring characters (paragraph 3).
Meaning is not described as something that overshadows character development.

71. C
A: No. Themes and patterns emerge for the writer only in retrospect; her stories “begin with the particular, never the
general” (paragraph 2).
B: No. This choice is too extreme. The author states that there are in fact hidden patterns and commonalities
(paragraphs 1, 2, and 3). She refers to these patterns as a way of discovering “sequence in experience” (beginning of
paragraph 1).
C: Yes. The author describes the “freedom ahead” at the start of each new story (paragraph 2). Patterns and themes
are not imposed on the story from the beginning; they only appear in hindsight.
D: No. While the author does suggest that she learns life lessons through her writing (paragraphs 1 and 2), she does
not suggest that the progression of one matches the other.

72. D
Note: The credited response will NOT have been described as a factor in creating a true work of fiction.
A: No. Memory is discussed in paragraph 4 as a contributing factor.
B: No. Experience is discussed in the same context as memory, as something that the author draws upon to create
fiction (paragraph 4).
C: No. The author states that she invents her characters, for example, rather than copying them from real people
(paragraph 4). This suggests the use of imagination.
D: Yes. The author does use bits and pieces of her own experiences in her work (paragraph 4). However,
“autobiography” indicates that the author writes fiction as an account or mirror of her own life, an undertaking
rejected by this author. In particular, she refuses to turn real people she has known well into characters in her stories.

73. D
I: True. In paragraphs 1 and 2, the author states that patterns in her work repeat themselves in a way visible only in
hindsight
II: True. One repeated pattern is the reappearance of characters from other stories (paragraph 3).
III: True. In the beginning of the passage, the author describes writing as a way of uncovering the action of cause and
effect in one’s life.

74. C
Note: The credited response will NOT be described in the passage as contributing to the development of a character.
A: No. Personal experiences do in fact contribute to character development, as attested to in para- graph 4.
B: No. Luck, while not the primary factor, does at times play a role in creating a character (paragraph 4).
C: Yes. This is the factor which, according to the author, does not play a role in character devel- opment. The author
claims that when while writing one story, no other stories “exist,” and that each writer must find meaning for him or
herself (paragraph 2).

LOGICAL REASONING (Sectional Test 4) PAGE 17


© Copyright 2020 @LEGALIGHT Education
D: No. The author refers to the plot as a contributing factor in the sentence, “Characters I invent along with the story
that carries them” (paragraph 4).

Passage 14
75. D
A: No. According to the passage, the idea originated during the first Congress of the United States (first sentence). It
was over 150 years later that the idea was revived by Republican Con- ventions (paragraph 2). This is the only mention
made in the passage of major political parties.
B: No. Term limits were first proposed during the first Congress (first sentence). State constitu- tional conventions are
not mentioned in this context.
C: No. The author does state that voters today support term limits (paragraph 2). However, the idea originated over
150 years before 1943 (paragraph 1) and citizen-driven referenda are not mentioned as a factor in those times.
D: Yes. The passage states that support for term limits first appeared during the first Congress (first sentence).

76. C
A: No. Compare this choice piece by piece to the passage, and to choice C. The passage ends with a prediction that the
states’ attempts to evade the issue of constitutionality will fail. It does not end with opposition to the likelihood that
the courts or Congress will negate state-based term limits.
B: No. There is no qualification at the end of the passage to the prediction that state-level term limits fwaiilll. Instead,
the passage closes with the claim that only limits imposed through a (national) Constitutional amendment could
survive.
C: Yes. The author describes current enthusiasm for term limits in paragraphs 1 and 2. At the end of paragraph 2, the
author predicts that limitations based on amendments to state constitutions will fail. Next, the author gives reasons
why he or she believes this prediction to be accurate (paragraphs 3 and 4). In paragraph 5, the author describes how
states have attempted to avoid negation of their term limits by the courts and Congress. Finally, in paragraph 6, the
author reasserts his or her claim that term limits not based on an amend- ment to the federal Constitution will be
overruled.
D: No. The author never revises his or her prediction (it is reasserted in the last paragraph). The author also does not
return at the end of the passage to a contrast between past and present.

77. C
A: No. The Supreme Court case is cited to illustrate a case made against term limits (paragraph 4), not to argue for the
wisdom of limits.
B: No. The Powell v. McCormack decision is not presented as flawed or mistaken.
C: Yes. The case is cited by opponents of term limits in order to argue that Article 1 of the Constitution does not allow
for additional requirements for office-holding (paragraph 4).
D: No. The Court in this case ruled that term limits were unconstitutional (paragraph 4).

78. B
Note: The credited response will NOT be given as evidence in support of the idea that support for term limits is
growing.
A: No. The Republican Conventions are presented as evidence that the idea of limits is “hotter than ever” (paragraph
2).
B: Yes. This contradicts the passage, which states that Congress is likely to negate any state laws imposing limits
(paragraph 6).
C: No. Voter support for term limits is mentioned in paragraph 2.
D: No. State legislation limiting successive terms is mentioned in paragraph 2.

79. C
A: No. The author argues that term limits that are not supported by an amendment to the Con- stitution (that is, those
imposed by the states), not all term limits, will fail (paragraph 6).
B: No. This choice is too narrow. It only describes the purpose of the fifth paragraph.
C: Yes. Throughout the passage, the author explains why term limits not based on a Constitu- tional amendment will
fail.

LOGICAL REASONING (Sectional Test 4) PAGE 18


© Copyright 2020 @LEGALIGHT Education
D: No. While the author argues that some term limits may currently be unconstitutional, the passage does not indicate
that the author opposes an amendment to the Constitution that would allow term limits.

Passage 15
80. D
I: True. The combination of opium and alcohol guaranteed “a genuine pharmacologic effect” that convinced
consumers that the medicines were effective for a wide variety of illnesses (paragraph 3). This effect would likely be
used in any sales pitch.
II: False. Opium taken orally does not produce sudden rushes of euphoria (paragraph 5). Thus, it would not be
marketed on that basis.
III: True. The passage explicitly states that patent medicines were sold as cures for opium addic- tion (paragraph 2).

81. D
A: No. The correct answer will list an unlikely characteristic of modern opiate addicts. In para- graph 6, the author
refers to many reported cases of opiate addiction among modern-day physicians. No reason is given in the passage to
exclude European physicians.
B: No. In paragraph 6, the author suggests that opiate addiction is not uncommon among physicians.
C: No. In paragraphs 5 and 6, the author suggests that the typical modern opiate addict is an intravenous heroin user.
D: Yes. The average housewife fits the turn-of-the-century profile (paragraph 5), not the mod- ern pattern of opiate
addiction.

82. A
A: Yes. In the fifth and sixth paragraphs the author argues that in the nineteenth century, opiate addiction usually did
not interfere in daily life, and did not involve physical danger. This contrasts sharply with the dangers of modern heroin
addiction.
B: No. Addiction to oral opiates is not described as a modern-day problem.
C: No. Oral opium and heroin are portrayed as being significantly different in both their physical and social effects.
There is no suggestion in the passage that lack of nineteenth-century regula- tion contributed to modern problems
with heroin.
D: No. Poor training of nineteenth-century doctors did contribute to the popularity of opiate- containing patent
medicines in the 1800s (paragraph 1). However, no connection is made by the author to modern problems with heroin
addiction.

83. A
Note: The credited response will describe a factor that did NOT, as far as we know from the passage, contribute to the
success of patent medicines.
A: Yes. The author argues that the cost of prescription medication played a role, but never sug- gests that physicians
were unwilling to prescribe opiates.
B: No. Lack of access to professional medical care and medication is given in the passage as a significant factor
(paragraph 1).
C: No. The author does in fact describe patent medicine as cheaper than prescription medications (paragraph 1).
D: No. The author indicates in paragraph 3 that people commonly believed the patent medicines to be effective and
rapid cures. The large number of people using medications containing opium in the nineteenth century (paragraph 4)
provides further evidence of how common this belief was.

84. D
Note: The correct choice will be a statement that would NOT be supported by the passage.
A: No. In paragraph 3 the author suggests that the addition of alcohol to opium ensured that there would be a
noticeable pharmacologic effect.
B: No. Because labeling of patent medicines was not required, patients had no way of knowing what the opium content
was (paragraph 2).
C: No. The author sharply contrasts addiction to oral opiates on one hand, and to heroin on the other. Heroin is closely
associated with AIDS and hepatitis (paragraph 6). From this we can infer that oral opiate use is not, according to the
passage.

LOGICAL REASONING (Sectional Test 4) PAGE 19


© Copyright 2020 @LEGALIGHT Education
D: Yes. This number corresponds to the per-capita importation of crude opium (paragraph 4), not to the number of
opiate addicts.

85. B
A: No. The passage never suggests that opiates in any form have true medicinal effects.
B: Yes. This is one of the contrasts drawn between oral and intravenous opiates (paragraph 5). “Euphoria” in the
passage corresponds to “mood swings” in the answer choice.
C: No. The passage suggests just the opposite—that oral opiate users are less vulnerable to vio- lence than intravenous
users (paragraph 6).
D: No. Neither oral nor intravenous opiates are claimed to be less addictive.

Passage 16
86. D
A: No. These substances are toxic to all life and cannot be used to treat cancer (paragraph 2).
B: No. There is no known selective (non-toxic to humans) treatment for cancer in either real or theoretical terms
(paragraph 6).
C: No. This is a misuse of the term “selective.” The author uses the word to refer to treatments (paragraph 2), not to
diseases or to bacteria.
D: Yes. The author asserts and explains the similarity in paragraph 6.

87. B
A: No. The cells of human beings have no walls (paragraph 4).
B: Yes. See the beginning of paragraph 4 and the end of paragraph 5.
C: No. Penicillin works by damaging cells with walls (paragraph 4).
D: No. The effectiveness of penicillin comes from what the drug actually does, not from our un- derstanding of what
it does. Penicillin was used effectively before we understood its mechanism (paragraph 5).

88. B
A: No. “Selective” indicates a characteristic of therapy, not a theory, and as a concept and a goal it is still very much in
use (paragraph 2).
B: Yes. Selective treatments are less damaging to the patient, and so selectivity would contrib- ute to the success of
the therapy (paragraphs 2 and 6).
C: No. AIDS is mentioned only in comparison to highlight the threat posed by cancer (para- graph 1). No specific types
of AIDS research are discussed.
D: No. “Selective” describes the treatment for disease (penicillin, for example), not the disease itself (paragraph 3).

89. D
A: No. According to the passage, a treatment that is poisonous to disease-causing cells, and not to normal human cells,
would be effective (paragraph 2).
B: No. Lack of action on bacterial cells would indicate less, not more utility.
C: No. Treatments that affect cell walls but not cells are never described. Penicillin is useful because it harms bacterial
cells by damaging their walls (paragraph 4).
D: Yes. Treatments are most useful if they are selective, meaning they harm disease-causing cells but not normal
human cells (end of paragraph 2).

90. C
I: True. The correct choice(s) will be inconsistent with the author’s description of the way in which penicillin works.
The author argues that penicillin works by damaging bacterial cell walls (paragraph 3).
II: False. Cure of a cancer patient by administering a poison would have no relevance to the author’s description of the
way in which penicillin works.
III: True. The author argues that bacterial cells are hyperosmotic relative to their surroundings, and that is why they
need complete cell walls (which penicillin destroys) to survive (paragraph 3). If it were shown to be true that penicillin
is effective against non-hyperosmotic cells, it would undermine the author’s explanation of the mechanism through
which penicillin works.

LOGICAL REASONING (Sectional Test 4) PAGE 20


© Copyright 2020 @LEGALIGHT Education

Passage 17
91. D
A: No. The author never indicates that the Shakers had limited resources with which to work.
They avoided ornamentation out of a belief in simplicity, not out of necessity.
B: No. Shakers valued order at one time but not at all times in their history (paragraph 2). Order is not presented by
the author as a reason for the simplicity of Shaker works.
C: No. Shakers believed in absence of decoration (paragraph 5), not that simplicity was more decorative.
D: Yes. The Shakers believed in “function, not extravagance” (paragraph 5).

92. B
Note: The credited response will NOT weaken Applebaum’s explanation.
A: No. This would weaken Applebaum’s claims by suggesting that the real reason for the Shakers’ ecstatic ceremonies
may have been the influence of a “broader national trend” of revivalism (see paragraph 4), not repressed sexuality.
B: Yes. This claim would be consistent with Applebaum’s argument that repressed sexuality was the cause. A
correlation between increased separation of the sexes (if the English Shak- ers did not segregate the sexes but the
American Shakers did) and the changes in Shaker ceremonies (after they migrated) would strengthen, not weaken, his
interpretation.
C: No. If the Shakers in reality were not especially chaste, there would be no reason to believe that their sexuality was
unusually repressed (paragraph 3).
D: No. Applebaum argues that ecstatic dancing may have been a way in which the sexually repressed Shakers
expressed the affection for each other that “less restricted people express in a more direct fashion” (paragraph 3).
However, if the Shakers were unaffectionate by nature, it would undermine this interpretation.

93. C
A: No. Shaker furniture was characterized by a lack of ornamentation. Therefore, one would not expect illustrations
and figurines on such a piece.
B: No. Shaker furniture was built to be functional (paragraph 5), which would entail a practical design.
C: Yes. Shaker furniture was designed “without ornamentation” (paragraph 5).
D: No. The issue of multiple uses for single pieces is never discussed.

94. A
A: Yes. Michelangelo’s painting is offered as evidence that the relationship between great art and faith acts as a general
principle, and that it is not limited to the Shakers (paragraph 6).
B: No. The author never claims that all great art is characterized by simplicity, nor does he or she connect simplicity to
deep faith in general terms.
C: No. The correct answer will go beyond the particular case of the Shakers and their art.
D: No. The correct response will show how the relationship between great art and deep faith ex- ists in cases other
than that of the Shakers.

95. A
A: Yes. The author states that great art is usually produced by people who have a deep faith (paragraph 6). Thus these
works would likely have been produced by people with great faith. The fact that they are elaborate is irrelevant; the
author does not argue that great art must always be simple.
B: No. The Shakers were energetic, but the author does not argue that energy and enthusiasm give rise to great art.
C: No. The Shakers were not especially sophisticated, and they produced great art. The author makes no causal
connection between great art and sophistication.
D: No. This is a characteristic specific to the Shakers. The author proposes deep faith, not sim- plicity, as a generator
of great art (paragraph 6).

LOGICAL REASONING (Sectional Test 4) PAGE 21


© Copyright 2020 @LEGALIGHT Education

Passage 18
96. D
A: No. This choice is too narrow. It is directly supported by the passage in the beginning of para- graph 2, but represents
only one piece of the author’s overall argument.
B: No. This choice is too narrow and is not consistent with the passage. The author asserts that openness and relativism
are taken as “moral postulates” (not theoretical insights) by the stu- dents (paragraph 1). The author himself does not
accept those beliefs as morally insightful.
C: No. This choice is too broad. While the author does posit this interrelationship as a general rule (paragraph 2), the
passage as a whole explores its meaning for one particular case—the United States.
D: Yes. The first two paragraphs describe why and how the educational system has come to inculcate relativism in its
students, and the final two paragraphs indicate how this could be destructive to the social contract and national
cohesion.

97. C
A: No. The word “all” makes this choice too extreme. According to the passage, this is true of “almost every student”
(first sentence).
B: No. The author identifies an almost unanimous belief in relativism (as opposed to belief that a particular truth stands
above all others) on today’s campuses. The students are not debating the issue; they for the most part accept it as
true without question (first half of the first para- graph).
C: Yes. See the first half of the first paragraph. According to the author, students have all been educated in the “moral
postulates” of relativity of truth and of openness. Most accept it without question, often without an understanding
that it could even be open to question.
D: No. The author states that openness is the only virtue that all early education teaches, not that it is the only virtue
taught in any school (paragraph 1).

98. A
A: Yes. The passage claims that the historical origins of the United States and the belief in natural rights bound up with
those origins are neither taught nor respected within the education of the “democratic personality” (second half of
the second paragraph). The quote provided in the question suggests that this disappearance of traditional beliefs (such
as those described in paragraph 2) may entail the loss of a source of individual strength of character.
B: No. This choice takes the word “petrified” out of context. The quote uses the word to com- municate the idea that
bits of wisdom from the past have hardened into lasting and “active” beliefs, carried on through the generations. This
answer choice is inconsistent with the quote’s portrayal of those beliefs as active, alive, and useful today. Don’t be
misled by the word “tradi- tion”—while the author argues in the passage that the moral standards of “traditional com-
munities” may not be relevant to a modern democratic state (paragraph 2), this does not apply to all traditions or
ideas from the past.
C: No. See the middle of paragraph 2. The author has no nostalgia for traditional communi- ties, which are based on
ideas and beliefs quite different from those of the “democratic man.” Rather, he regrets the waning of the tradition
of natural rights as it is expressed in the Consti- tution and Declaration of Independence.
D: No. The author criticizes relativism, or the belief that one must remain open to and accepting of all ideas and values
held by others.

99. B
A: No. A demonstration that the Constitution did not truly represent a shared belief in equality and liberty would not,
by itself, significantly undermine the author’s claim that such shared visions are generally necessary for national unity.
B: Yes. The author argues that the education of the “democratic man” was founded on belief in and attachment to
basic principles (the rights doctrine) embodied in the Constitution (middle of paragraph 2). If the Constitution did not
in fact embody some shared vision of
rights and liberty, it would undermine the author’s claim that this education involved a com- ing together in the belief
in certain basic truths to be held above all others.
C: No. This choice misrepresents the author’s views. He argues that Americans in the past were committed to certain
particular beliefs, rather to a belief in openness (relativism) to all points of view (middle of paragraph 2).

LOGICAL REASONING (Sectional Test 4) PAGE 22


© Copyright 2020 @LEGALIGHT Education
D: No. “Disprove” is too strong a term. The information in the question would erode the author’s picture of unanimity
in certain beliefs in early American history, but it in and of itself is not enough to disprove his overall claim about the
evolution of the educational system.

100. D
A: No. The author makes no reference to explicit legal codes as constituting the social contract.
B: No. This choice takes words out of context of the passage. The American social contract is (or was) based in part in
a coming together in the belief that we are all created equal (paragraph 2), but the contract itself is not reducible to a
belief in equality.
C: No. The social contract is based on belief in the ideas expressed in the Constitution. The con- tract is not the actual
document itself.
D: Yes. The author fears that relativism, or lack of agreement on certain fundamental truths, will weaken or destroy
the social contract (paragraphs 3 and 4). The social contract is not a written code or document, but a willingness to
compromise some of one’s individual beliefs in the service of a “shared goal or vision of the public good” (end of
paragraph 3).

101. B
A: No. The democratic man was formed by dedication to “a form of government and its rational principles” (second
half of paragraph 2), not to a particular political party.
B: Yes. The democratic man was educated to believe in the natural rights and principles embodied in the Constitution,
the Declaration of Independence, and in the history of the United States (paragraph 2). The democratic personality is
educated in relativism, the belief that no one belief is more “true” than another (middle of paragraph 1).
C: No. The democratic man believes that “all men are created equal” (middle of paragraph 2). This is not a distinction
between the two. The point of contrast is that the democratic man does not accept that all beliefs or principles are
created equal.
D: No. The democratic man was formed with an attachment to the “letter and spirit of the Dec- laration of
Independence,” that is, with loyalty to certain ideas and to a type of government (paragraph 2). The author contrasts
this education with “the kinds of attachment required for traditional communities,” which include passionate
patriotism and unthinking loyalty to a particular country.

102. C
A: No. This statement is too extreme. The author suggests (through his discussion of the relativ- ist view of the past at
the end of paragraph 1) that he believes we should correct our historical mistakes and “really be right.” He clearly
wishes to celebrate the historical origins of the na- tion (paragraph 3), but never suggests that past mistakes should
be hidden or forgotten.
B: No. He identifies the ideas in these documents as a source of the American social contract and national unity
(paragraphs 3 and 4). He does not, however, indicate that all national commu- nities should be defined through similar
foundational documents.
C: Yes. The author views our history as a source of identity and unity (first half of paragraph 2), and indicates that we
should also learn from it in order to correct our mistakes and “really be right.” This is in contrast to the relativists who,
according to the author, wish to disas- sociate from our past. In the author’s view, relativism teaches that we should
not hold or defend true beliefs in the first place, as opposed to implementing our true beliefs by learn- ing from and
correcting our mistakes.
D: No. This choice presents the author’s depiction of relativist beliefs, not the ideas held by the author himself (end of
paragraph 1).

Passage 19
103. B
A: No. The author of the passage claims that poems need no defending (paragraph 9), and ex- presses no particular
affection for indefensible poems.
B: Yes. The author has warm feelings for young poets, in part because they are in love with writing (paragraph 3). As
for the poem, the author indicates that great poems communicate passion and vision (paragraph 5).
C: No. The author uses a critical tone in describing poets who are unwilling to discuss or explain their work (paragraph
7), and directly states that he or she dislikes indecipherable poetry (paragraph 3).

LOGICAL REASONING (Sectional Test 4) PAGE 23


© Copyright 2020 @LEGALIGHT Education
D: No. The author expresses warmth towards young (indecipherable) poets (paragraph 3), while never discussing
feelings about mature poets. As for the poem itself, the author says nothing to suggest that he or she particularly
values youthful and optimistic poetry.

104. D
A: No. While the second paragraph indicates that poets hope to be praised, it would be too ex- treme to infer from
this that poets as a rule dislike thoughtful criticism once it is given.
B: No. If young and inexperienced poets tend to feel secure, then there is no particular reason they should react with
anger when their work is critiqued.
C: No. The author never describes young or inexperienced poets as being especially liable to take critiques personally.
If anything, they are portrayed as relatively thick-skinned in the face of criticism, in most situations (paragraph 7).
D: Yes. According to the passage, inexperienced poets believe that if their poetry is difficult to understand, it
approaches greatness (paragraphs 5 and 6). If a critic were able to understand the poem, the work may not be quite
as difficult as intended and so not as great. Thus the author specifically suggests indecipherability or lack thereof as a
potential sore point for inexperienced writers.

105. B
A: No. The correct answer will present a situation in which a person makes communication im- possible through the
use of some form of private language. Braille is a public language—just because the young woman has not learned it
does not mean that the language itself is indeci- pherable.
B: Yes. The stenographer takes what is usually a form of communication, and renders it indeci- pherable to others
through the use of abbreviations only she could understand.
C: No. The individual student’s lack of comprehension does not constitute evidence that the formulas are
fundamentally incomprehensible to others.
D: No. This is an example of clear communication, not of indecipherability.

106. D
A: No. The author does not claim that only great poems arise from or communicate passion. B: No. The author’s claim
does not depend on there being a physiological connection.
C: No. The author does not argue that passion always produces great poetry; the claim is that great poetry always
evokes passion.
D: Yes. The passage suggests that Pound is a great poet (paragraph 6). If his poetry fails to express passion, this would
weaken the author’s assertion that passion is necessary for great poetry.

107. B
A: No. This statement is made as part of the author’s discussion of the defense of poems later in the passage
(paragraph 9). Frailty has no direct connection to the novice’s belief in his or her own immunity.
B: Yes. Paragraph 7 illustrates the circularity in the logic of young poets. If their poems are criticized, that is simply an
indication of lack of understanding on the part of the critic.
After all, if they understood the poem, they would not criticize it. With this kind of logic in hand, the poet can write
anything, no matter how obscure or awkward, and claim that it is just as it should be.
C: No. The intelligence and likeability of the novice poet (paragraph 3) does not explain why he or she sees their work
as impregnable.
D: No. The author makes this statement to illustrate the yearning for praise that underlies a poet’s plea for brutal
critique (paragraph 2). It is not directly relevant to the illusions of invul- nerability attributed to young poets in
paragraph 7.

108. C
A: No. While the author’s description of young poets suggests that some may be insecure, the passage does not
suggest that poetry arises from or is shaped by insecurity.
B: No. A poem may be innocent or “blameless for its content” (paragraph 1), but thisaischarac - teristic, not a
cause, of poetry.
C: Yes. In paragraph 1 the author states that a poem “usually represents its author’s pride.”

LOGICAL REASONING (Sectional Test 4) PAGE 24


© Copyright 2020 @LEGALIGHT Education
D: No. The critic never feels brutal towards a poem (paragraph 1), but doesn’t describe poetry that itself manifests or
is created by brutality.

Passage 20
109. D
A: No. The correct answer will be a statement with which one group will agree, and the other will disagree. The author
states that “every known language first appeared in spoken form” (paragraph 3), and gives no indication that the early
grammarians would contest this fact. Thus it seems that both early and modern would disagree with this statement,
not disagree with each other over it.
B: No. The author states this in reference to the historical beginnings of traditional grammar. There is no evidence that
either the early or modern grammarians would disagree with it.
C: No. Again, this is part of the author’s description of the history of the study of grammar. It is not presented as a
point of debate between early and modern grammarians.
D: Yes. Early grammarians study written texts, and so would say that punctuation is more im- portant than gesture.
The modern grammarians, who focus on spoken language, would say yes, gesture is just as important (paragraph 3).
Thus the early and modern would disagree over this statement.

110. A
A: Yes. According to the modern or current view, this is true both of individuals (paragraph 3) and of languages
themselves.
B: No. Expression in written speech requires punctuation (paragraph 4), and nothing about punctuation is given as a
reason for the primacy of spoken language.
C: No. Precision is not discussed in the passage.
D: No. The contribution of speech to literature is never discussed.

111. D
A: No. This choice is too extreme. People and animals speak different languages (paragraph 6), but that by itself is not
enough evidence that we are completely unable to communicate in any form.
B: No. Animals may speak inferior languages (paragraph 6), yet that does not mean that they cannot truly
communicate with each other. This choice is too extreme.
C: No. The languages of animals may be inferior to those of humans (paragraph 6), but this indicates nothing about
their need to communicate.
D: Yes. The author argues that the “open-endedness” and thus the superiority of human lan- guages contributes to
our position on earth (paragraph 6). By those terms, if our language is superior, our position must also be superior.

112. A
A: Yes. In the sentence cited, the author indicates that most modern linguists take it as true that speech is primary.
Thus we can infer that “axiomatic” in this context means that they take or follow that belief as if it were a rule.
B: No. The passage indicates just the opposite. “Axiomatic” is used to indicate that modern lin- guists take primacy of
speech to be a rule, not an exception to some other rule.
C: No. This choice is too extreme. Most, not all linguists agree. Furthermore, some qualifications on the derivation of
written from spoken language are discussed (paragraph 4). Thus the word “perfect” goes too far.
D: No. The passage uses “axiomatic” to mean a point of agreement. The word itself indicates nothing about an
innovative or cutting-edge quality of the belief in primacy of speech.

113. C
I: False. Inflection or lack thereof in written speech is never discussed in the passage. Be careful not to answer based
on common sense or outside knowledge.
II: False. The author never discusses lack of opportunity for tone in written speech.
III: True. According to the passage, written punctuation takes the place of the gestures and facial expressions that
carry meaning in face-to-face speech.

LOGICAL REASONING (Sectional Test 4) PAGE 25


© Copyright 2020 @LEGALIGHT Education

Passage 21
114. B
A: No. The author specifies a variety of reasons that modern listeners may fail to appreciate con- temporary music.
Thus the passage suggests that people today criticize modern music for those specific characteristics, not because they
believe it to be insufficiently universal. The author does use the word “universal” in paragraph 5, but in the context of
the claim that contempo- rary music should be listened to because of its unique qualities.
B: Yes. In paragraph 3 the author says that avoidance of sentiment and feeling is the “reproach that is repeated more
often than any other.”
C: No. The author states that modern music does express subtleties of light and dark (paragraph 6). D: No. Lack of
straightforward melody is one potential problem or source of confusion (paragraph 2), but it is not the most common
problem. That problem would be the “reproach repeated more
often than any other,” the music’s apparent lack of sentiment and feeling (paragraph 3).

115. A
A: Yes. The passage asserts that repeated hearings of intricate melodies will clarify their appeal (paragraph 2).
B: No. The author recommends listening repeatedly to the same piece (paragraph 2), not aban- doning it for another
piece.
C: No. In paragraph 2 the author suggests listening to the piece again, not casting it aside for a different piece and
different composer.
D: No. The author gives explicit advice; listen to that piece repeatedly until it becomes clear.

116. D
A: No. The correct answer will be a statement made by the author that is inconsistent with the scenario presented in
the question. The author’s assertion that older music should seem foreign to us has no direct relevance to the case of
the counterfeit music.
B: No. Controversy and resentment are not issues here.
C: No. The scenario describes a piece in the style of the sixteenth century; it suggests nothing about sensitivity to
contemporary music.
D: Yes. The question describes an exact duplication of the sixteenth-century mode of expres- sion; the author claims
in the statement cited in the answer choice that such duplication is impossible (paragraph 6).

117. D
A: No. We would expect to find such climaxes in modern music (paragraph 6). Milhaud and Schuman are presented as
modern composers whose music should strike us as less foreign than that of the older (deceased) composers
(paragraph 5).
B: No. We would expect to find abandonment and hysteria in modern composers such as Bartók (paragraph 4).
C: No. Sessions and Bartók are shown to be modern composers through the author’s contrast between them and
Brahms and Tchaikovsky (paragraph 4). Thus it would not be surprising to discover such climaxes and explosions in
their work.
D: Yes. Buxtehude and Cherubini are composers of the past (paragraph 5). Therefore it would be surprising to find
climaxes of abandonment and explosive hysteria, characteristics of contemporary music, in their compositions
(paragraph 6).

118. B
A: No. The author argues that modern music is or can be just as enjoyable as romantic composi- tions, not that
romantic music is just as good as contemporary works.
B: Yes. The author argues that we do not expect the modern Eliot to write with the voice of the romantics Hugo and
Scott, and makes an analogy to what we should not expect of contem- porary music (paragraph 4).
C: No. The author mentions Eliot and Hugo to draw an analogy, not to dramatize differences between literature and
music (paragraph 4).
D: No. The author suggests that Eliot’s work is different from that of writers of the past, and so should be appreciated
on its own terms. He uses this claim to make the point that listeners who complain that modern music is too dry and
cerebral simply do not sufficiently appreciate the characteristic musical speech of their own time (paragraph 4).

LOGICAL REASONING (Sectional Test 4) PAGE 26


© Copyright 2020 @LEGALIGHT Education
119. C
Note: The credited response will be one that either strengthens or has no effect on the author’s contention.
A: No. This choice would weaken the passage by indicating that romantic standards of com- parison have relevance to
contemporary music; the author himself recognizes a valid analogy between music and literature (paragraph 4).
B: No. This choice weakens the analogy made by the author between readers’ appreciation of modern writing and the
appreciation listeners should have for modern music (paragraph 4). If contemporary readers prefer romantic works,
the author’s contention (that those who fail to appreciate modern music are using inappropriate standards of
comparison) becomes less convincing.
C: Yes. This choice would have no effect on the author’s contention. The fact that Eliot studied works from the past
does not indicate that he followed their stylistic models or that contem- porary literature does not have its own unique
language.
D: No. This choice directly contradicts the author’s claims that each period has its own unique form of expression
(paragraph 6), and that we cannot judge contemporary music and litera- ture by romantic standards (paragraph 4).

Passage 22
120. D
A: No. The correlativity thesis indicates that if we do show that we have obligations to animals, animals must have
corresponding rights (paragraph 5). The author has us suppose that we have such obligations for the sake of explaining
the thesis, but the passage never indicates that we do in fact have obligations to animals.
B: No. The opponents of the thesis argue that it is invalid because objects and resources have no rights (paragraph 6).
It is the proponents of the thesis who raise the issue of direct vs. indirect duties (paragraph 7).
C: No. The author never states a personal position on the question of whether or not animals have rights.
D: Yes. If it were to be demonstrated that we have obligations to animals, and the thesis itself is accepted, it would be
true that animals have rights (paragraph 5). Notice the moderate wording of this answer choice compared to choice
A.

121. B
A: No. The correct answer will be inconsistent with Aquinas’s views as they are described in the passage. According to
the author, Aquinas believes that we have no duties to animals; he never indicates that God does not love them.
B: Yes. Paragraph 3 tells us that Aquinas believes that we sin only when we fail to fulfill a duty to ourselves or to God,
and that we have no duties to animals. The only way slaying of ani- mals and plants would be a sin in these terms is if
it leads us to sin against rational beings. The quote presents an analogy between humans on one hand and animals
and plants on the other by claiming that taking the life of any plant or animal is a sin “for the same reason” (deprivation
of life) that killing a human is a sin. This equivalence drawn between humans and animals is inconsistent with Aquinas’s
views as they are presented in the passage.
C: No. This statement is not inconsistent with Aquinas’s view that our treatment of animals is to be judged by its effects
on humans (paragraph 4).
D: No. Aquinas agrees that animals are irrational (paragraph 4), and the passage never indicates that he supported
human friendship with them. This choice is not inconsistent, and so does not undermine the author’s depiction.

122. C
A: No. All we know of Descartes from the passage is that he believed (unlike Aquinas and Aris- totle) that animals are
not sentient (paragraph 3). Aquinas would disagree with the statement that we have obligations to animals, but we
do not know Descartes’ position on the subject.
B: No. As in choice A, we only know of Descartes’ position on sentience. We cannot infer any- thing from that about
his position on reason.
C: Yes. The passage defines sentience as “the capacity to experience pleasure or pain” (para- graph 2). Thus Aquinas,
who unlike Descartes believes animals are sentient (paragraph 3), would agree with the statement and Descartes
would disagree.
D: No. This answer choice applies to the discussion of correlativity in the final two paragraphs. No information is given
in the passage to allow us to deduce the position of either Descartes or Aquinas on that issue.

LOGICAL REASONING (Sectional Test 4) PAGE 27


© Copyright 2020 @LEGALIGHT Education
123. D
A: No. The author defines sentience as “the capacity to experience pleasure and pain” (paragraph 2). The quote from
Sartre entails how man acts to define or create himself in the world—there is no connection in the quote or in the
passage between feelings and actions.
B: No. The correlativity thesis (if we have duties to others, they have corresponding rights (para- graph 5) has no direct
relevance to this quote.
C: No. The issue of direct vs. indirect duties arises in the context of discussion of the correlativity thesis (paragraphs 5
and 7), which has no relevance to the Sartre quote.
D: Yes. The author defines autonomy as “the capacity to make free choices” (paragraph 2). In the quote given in the
question, Sartre describes how man makes and defines himself through his free will: “he will be what he makes
himself” or what he chooses to be.

124. D
A: No. The correlativity thesis, as described here, makes no claims or assumptions based on equal rights among animals
or between animal species.
B: No. Proponents of the thesis argue that we have an indirect, not a direct duty to things such as oceans and
woodlands—our direct duty is to future generations themselves (paragraph 7).
C: No. The proponents’ position indicates that we have a direct duty to future generations (para- graph 7).
D: Yes. Critics of the thesis claim that we have a duty not to destroy certain things that clearly do not have
corresponding rights (paragraph 6). The proponents respond that we have only indirect duties to such things, and that
the thesis applies only to cases of direct duty (para- graph 7). If we were to have a direct duty to future generations to
preserve the ecosystem, it would support the distinction drawn by the proponents. It would also further undermine
their critics’ supposed counter example by showing that correlativity could explain our duty to preserve resources
without claiming that those resources somehow themselves have rights.

125. D
A: No. Descartes is mentioned only in passing as someone who holds a different position on sen- tience (paragraph 3).
The author does not dwell on or highlight this difference.
B: No. The author does mention that the idea that a being must be able to reason in order to have rights goes back to
Aristotle (paragraph 2). However, sentience and reason are discussed in the second paragraph in order to introduce
the author’s discussion of Aquinas’s views, a discussion which continues on into the third and fourth paragraphs.
Furthermore, sentience and reason are presented as necessary, not sufficient conditions. We have no duties towards
beings that lack them (paragraph 2), yet the passage never indicates that we do have duties towards all be- ings that
satisfy those conditions.
C: No. The author never expresses a personal opinion on the issue of animal rights. Furthermore, sentience and
rationality are part of the author’s discussion of duties. His consideration of whether the existence of duties would
entail the existence of rights comes later in the passage, in the final two paragraphs.
D: Yes. The author raises the issues of sentience and rationality in order to introduce the discussion of Aquinas’s claim
that we have no duties towards animals because they, while sentient, lack the capacity for reason (paragraph 2).

126. B
A: No. This statement is suggested as a belief held by Aquinas (paragraph 4). The author never indicates that he agrees
with Aquinas on this point.
B: Yes. In the first paragraph the author states that the satisfaction of certain conditions may cause us to have duties
towards beings, including animals. Note that the author never states his own opinion on whether or not those
conditions are fulfilled by any non-human being.
C: No. This is taking the author’s statement in the beginning of paragraph 5 too literally. The author has us suppose
that we do have such an obligation in order to explain the correlativity thesis, not in order to indicate his own beliefs
on the topic.
D: No. The author describes others’ beliefs that we have a duty to preserve great art (paragraph 6).
However, we cannot infer that the author follows this belief. Note the wording used: “some argue” and “our supposed
duty.”

LOGICAL REASONING (Sectional Test 4) PAGE 28


© Copyright 2020 @LEGALIGHT Education

Passage 23
127. B
A: No. The author’s description of the curve for mesothelioma in paragraph 7 would apply to PML as well. The mortality
curve cannot be symmetrical. The lower boundary must be zero,
as the disease could at the latest be discovered at or soon after death (paragraph 7). However, a few individuals might
live for many years after diagnosis, thus skewing the curve to the right.
B: Yes. See paragraph 7. The discovery of the disease begins the curve at zero. According to the author, one could die
at the same time that the disease is discovered, but no earlier. How- ever, while half those diagnosed would die before
the median point and half after, some people may live much longer than four to six months after the median (that is,
longer than eight to twelve months total). If one person, for example, lived for several years (a likely pos- sibility), the
curve would extend far to the right to include that individual. Thus the curve would be skewed to the right.
C: No. The author states that disease is discovered at death or before (paragraph 7). Thus we can- not have a few
individuals whose disease is “discovered” long after death pulling the curve to the left in the same way that at least a
few long-term survivors are likely to pull it out to the right.
D: No. There is no evidence given in the question of the passage to indicate that the mean (average) mortality for PML
will be lower than the median (the middle number in the series) length of survival.

128. B
A: No. This choice is too wishy-washy to significantly strengthen the author’s claim. The third paragraph proposes a
possible causal connection between attitude and ability to fight off cancer. The fact that not all people with sanguine
or cheerful personalities get cancer does not show that their personality played any causal protective role.
B: Yes. This indicates that when a person’s attitude improves, they are less likely to die from ovarian cancer, thus
supporting the existence of a causal connection. Notice the word “sig- nificantly,” which makes this choice strong
enough to bolster the author’s claim.
C: No. This statement would somewhat weaken, not strengthen the passage. The author argues in paragraph 3 that a
positive attitude may boost the immune system and improve one’s abil- ity to fight cancer. This answer choice indicates
that the immune system has little to do with whether or not we have cancer.
D: No. This statement would weaken, not strengthen the author’s argument that a positive at- titude or emotional
status can help a person to resist cancer (paragraph 3).

129. C
A: No. Mesothelioma has a median mortality of eight months (paragraph 2). A median is the midpoint in a series,
meaning half the people die before eight months, and half after (para- graph 7). Therefore a majority will have died
after eight months, not within six months.
B: No. While the author suggests that most people would misunderstand or misinterpret the sta- tistical information
in the literature (paragraph 4), he does not suggest that misinterpretation of their own life expectancies constitutes
irresponsibility.
C: Yes. In paragraph 4, the author claims that the majority of people faced with these statistics would (incorrectly)
believe that they will die within eight months.
D: No. The author never suggests that people either can or will change their personality.

130. D
I: True. The author states that Platonic logic sees means and medians as “hard ‘realities’,” yet that in fact variation is
the true “reality” (paragraph 5).
II: True. The author would reject a Platonic quest for a hard and clear marker of the boundary at which life begins
(paragraph 5).
III: True. Platonic thought holds that clear distinctions and categorizations define “reality,” while the author believes
reality lies in variation (paragraph 5).

LOGICAL REASONING (Sectional Test 4) PAGE 29


© Copyright 2020 @LEGALIGHT Education

Passage 24
131. B
A: No. The Jellyby phenomenon occurs when people try to fix the problems of others while ignor- ing problems at
home (paragraph 5). The policeman may be working to solve the problems of others, but there is no indication that
he is failing to help people in his own neighborhood.
B: Yes. A psychiatrist tries to help others, but at the same time this psychiatrist is ignoring his own problems. Out of
these four choices, this one is the closest to the passage.
C: No. Neither solving the problems of others nor ignoring one’s own problems is involved in this example.
D: No. The employer is helping others in his own business, and there is no reason to conclude that he is ignoring
problems in his own home.

132. B
A: No. “Ideologues” in this context refers to European feminists. The author does not suggest that these feminists are
politically to the right.
B: Yes. This paragraph continues the author’s discussion of the irrelevance of European femi- nist ideas to Zimbabwean
reality. The “ideologues” are the European feminists who are lecturing the African women.
C: No. The ideologues are the European feminists who are being mocked or criticized by the Zimbabwean women.
D: No. The ideologues are Europeans, not Africans.

133. C
Note: The correct answer will be a statement that is inconsistent with the decisions described in the question.
A: No. Respecting Zimbabwean tradition is not inconsistent with focusing on the problems of others rather than the
problems in one’s own country. Furthermore, it doesn’t undermine the author’s claim that the phenomenon itself
arises out of the colonial experience.
B: No. There is no reference to attitudes towards the past in the new scenario described in the question.
C: Yes. If the majority of feminists agreed on a unified political agenda, it would undermine the author’s claim that
political groups are unable to work together (paragraph 1).
D: No. Perhaps they might find something to value in the future if feminists do come to be more responsive to their
concerns, but this does not mean that they currently see value in feminist ideas.

134. A
A: rYtes. Pa

of political movements’ intolerance of the past is the belief that they are doing everything for the first time, even when
this is clearly not the case (paragraph 1).
B: No. This is taking Larkin’s statement that “we invented sex” a bit too literally.
C: No. As in choice B, this takes Larkin’s statement to mean that people in the ’60s really did in- vent or transform sex
in some fundamental way. The author intends the statement to indicate just the opposite. Despite the beliefs of many
a political movement “in the full flush of its star” (paragraph 1), the author suggests that there is really nothing new
under the sun.
D: No. The author never refers to left or right-wing movements.

135. A
Note: The correct answer will be a question that cannot be answered by information provided in the passage.
A: Yes. While the author tells us that some African women raise their families on between sixty and eighty dollars a
month (paragraph 3), she never divulges the average income of African families.
B: No. The author asserts that we “think in terms of going out and telling the neighbors how to live” because we
(meaning Europeans) belong to “ex-colonizing countries” (paragraph 5).
Therefore, this “Jellyby phenomenon” affects how we perceive the world and our actions within it. Therefore, political
history (in this case colonialism) affects our perceptions.
C: No. In paragraph 1 the author states that the feminist movements of the ’60s were born in politics, as were the
feminist movements that arose from the French and Russian Revolutions. The rest of the first paragraph and the

LOGICAL REASONING (Sectional Test 4) PAGE 30


© Copyright 2020 @LEGALIGHT Education
second paragraph go on to list a variety of characteris- tics that all political movements, including feminist movements,
have in common.
D: No. This question is answered in the description of Zimbabwean women singing, dancing, and acting out their
complaints and criticisms (paragraph 4).

Passage 25
136. C
A: No. The theory of complementarity says that fundamental entities in some circumstances act
as particles and in others as waves, but that no experiment will show them acting in both ways at once (paragraph 4).
This experiment would contradict the theory, not prove it to be true.
B: No. Heisenberg’s uncertainty principle says that the more we know about an electron’s particle properties, the less
we know about its wave properties; no experiment shows an electron behav- ing as both at the same time (paragraph
4). Thus the answer choice would undermine, not support the uncertainty principle.
C: Yes. The Copenhagen interpretation (paragraph 6) is based in part on the principle that it is impossible to measure
position (characteristic of particles) and momentum (characteristic of waves) simultaneously (paragraphs 3 and 4). An
experiment that was able to detect both would call into question the principles of complementarity and uncertainty,
and so under- mine the Copenhagen interpretation which is based in part on those principles.
D: No. An experiment that detected both waves and particles would call into question the ideas of uncertainty and
complementarity (paragraphs 3 and 4), and so undermine the Copenhagen interpretation (paragraph 6).

137. C
A: No. Bohr and Born together developed the rules and philosophy of quantum cookery (para- graph 1). Their ideas
were entirely consistent, according to the passage.
B: No. Born focused on both particle and wave properties.
C: Yes. The author states that, somewhat paradoxically, experiments in quantum physics are rooted in classical physics
(paragraph 5).
D: No. Bohr tells us that observation interferes with the behavior of the atoms, and that “it is meaningless to ask what
atoms are doing when we are not looking at them” (paragraph 5). Born and Bohr are in complete agreement on all
points, as described by the passage.

138. C
A: No. This choice is too extreme. While the Copenhagen interpretation may be “slippery” (para- graph 6), the passage
never indicates that it is not susceptible to proof.
B: No. It has four parts: uncertainty, complementarity, probability, and disturbance of the system being observed by
the observer (paragraph 6).
C: Yes. The author claims that it may be used to solve problems even without an understanding of its basic principles
(paragraph 6).
D: No. The author describes it as “slippery,” and as being many things to many people (para- graph 6).

139. D
A: No. The passage states that both qualities cannot be measured at once (paragraph 3), not that entities cannot
express both at once.
B: No. Position is a property of particles, and momentum characterizes waves (paragraph 4).
Furthermore, while the passage states that both cannot be measured simultaneously, we don’t know that entities
cannot express both at once.
C: No. Position is a particle property, while momentum is a wave property (paragraph 4).
D: Yes. All parts of this choice are consistent with the description of Heisenberg’s principle in the third and fourth
paragraphs.

140. B
Note: The correct answer will be inconsistent with the Copenhagen interpretation.
A: No. The discovery of a particle that expressed more than one behavior would be consistent with the idea of
complementarity (paragraph 2), which is one of the four tenets of the interpre- tation (paragraph 6).

LOGICAL REASONING (Sectional Test 4) PAGE 31


© Copyright 2020 @LEGALIGHT Education
B: Yes. The principle of complementarity bases itself on the idea that fundamental entities ex- press both wave-like
and particle-like behaviors (paragraph 2). The discovery of a subatomic particle with no wave-like behavior would cast
doubt on the principle, and so also on the Copenhagen interpretation (paragraph 6).
C: No. This is consistent with the idea that the system under observation is disturbed by the observer (paragraph 5).
D: No. Born states that, given the problem of observation, all we can do is calculate the probabil- ity of a particular
result (paragraph 5). A particle that behaved consistently with such calcula- tions would add support to the
Copenhagen interpretation.

141. A
A: Yes. This is stated by the author in paragraph 5.
B: No. According to the passage, there is no such thing as absolute truth in quantum physics (paragraph 3).
C: No. This choice is too extreme. While quantum physicists attempt to calculate the probability of particular results
(paragraph 5), the author does not suggest that they are always successful.
D: No. The author refers to quantum cookery as “practical quantum physics since the 1920s” (first sentence). This
wording indicates that some aspects of quantum physics predated the 1920s; those aspects could well include some
form of experimentation. Similar wording with the same implication appears in the beginning of paragraph 3.

142. D
A: No. The uncertainty principle states that we cannot measure wave and particle functions si- multaneously
(paragraph 3). However, we can measure one or the other, suggesting that usable results are certainly attainable.
B: No. Chemical physics is never mentioned.
C: No. Quantum physicists must rely on probabilistic interpretations, given the problem of obser- vational interference
(paragraph 5).
D: Yes. The author writes that competent scientists can get useful results without any “great need for thought about
the fundamentals” (paragraph 6).

LOGICAL REASONING (Sectional Test 4) PAGE 32

You might also like